Uncommon Descent Serving The Intelligent Design Community

What is knowledge? (A response to Popperian)

Share
Facebook
Twitter
LinkedIn
Flipboard
Print
Email

ID debates often bring up foundational worldview issues, and the following exchange in the current Answering P thread is also worth headlining:

___________________

P, 62: >>Knowledge is information that, when embedded in a storage medium, plays a causal role in it being retained. This includes books, genomes and yes, brains. Furthermore, knowledge is objective in that is is independent of anyone’s belief. So, while I would agree that merely having a belief doesn’t make it true, we have a reason to suppose that our brains can genuinely contain knowledge.

What explanation do you have for the growth of knowledge? Let me guess: the reason why our beliefs may be true is because “that’s just what God must have wanted”?>>

KF, 64: >>Popperian:

Knowledge is information that, when embedded in a storage medium, plays a causal role in it being retained.

Gross error of definition.

Knowledge is not stored useful information but well warranted, credibly true belief. That is, responsibly free rational reflection is a condition of knowledge.

Knowledge comes about by a process of reflection, involving interior life and typically external experience and perception. This leads to processes of warrant on exertion of logical and/or empirical tests that lead to sufficient weight of warrant to put the stamp of credibility, knowledge, on. Though, this often comes with the proviso, that the degree of warrant is provisional, such as in science, management, law and many other practical affairs.

You have again set up and tried to knock over a strawman.

Now, of course, knowledge may often be posed in verbal forms or other representations amenable to storage, but that is secondary to what knowledge is.

Error, by contrast, often claims to be knowledge but in the end fails the test of warrant. (Let me add, following Aristotle, that truth says of what is that it is, and of what is not, that it is not.)

Knowledge is one form of reasonable and trustworthy faith.

And, I deliberately use this term to underscore that faith and reason — contrary to commonly seen skeptical notions — are not opposites that are inevitably at war with one another.>>

Box, 65: >>Popperian #62 claims to address KF’s point directly, however he does no such thing. KF’s point is about the process of rational inference. KF clearly shows that this process cannot be a chemical/mechanistic process.

Popperian again fails to address KF’s point.>>

Mung, 66: >>Popperian;

Knowledge is information that, when embedded in a storage medium, plays a causal role in it being retained.

And thus the pre-requisites for knowledge are … ?

And the knowledge to construct and integrate the required components for knowledge came from … ?

Since Popper rejected a causal theory of mind, what did he offer in it’s place?>>

C, 68: >>kairosfocus:

Knowledge is not stored useful information but well warranted, credibly true belief. That is, responsibly free rational reflection is a condition of knowledge.

I don’t think that’s the case when we humans speak of knowledge.

Just look at this debate.

If we take knowledge as being something we agree with after rational reflection, then knowledge starts to become a point of view.

For instance, at one time it was said that light travels in a straight line but it has been proved that isn’t the case.

Was the fact that light traveled in a straight line knowledge or an opinion?>>

C, 70: >> . . . The dispute is whether what we call “knowledge” changes.

Please try to understand the analogy.

What was considered “knowledge” was “error” in this case.

At one time, our pool of “knowledge” included the fact that the Earth was only a few thousand years old.

That “knowledge” has been replaced by the new “knowledge” that the Earth is billions of years old.

That previous “knowledge” was an “error”.>>

KF, 71: >>Carpathian,

do you consider scientific or historical knowledge, etc. to be knowledge?

If so, you are forced to a weak form view of knowledge that does not demand absolute certainty or incorrigibility. That is, the relevant degree of warrant for many fields of responsible practice or prudent behaviour is some type of moral certainty.

In sum, if X is warranted to this degree, it would be irresponsible or foolish to act as though it were false, never mind that you are open to possibility of correction. In short, knowledge in this sense is a certificate of reasonable, responsible trustworthiness. Not, a guarantee that what we think we know at any given point is beyond possibility of needing correction.

If instead you insist on absolute certainty, the field of knowledge would collapse to a very sparse set indeed. In particular, science, history, jurisprudence, economics and a good slice of mathematics post Godel’s incompleteness theorems would collapse.>>

____________________

Yes, what is knowledge lurks under the debates on ID. END

Comments
Popperian Apparently you are unfamiliar with Plato's discussion of and response to evolutionary materialism in his day in The Laws, Bk X. I suggest you read it. (added: as in immemorial already c 360 BC.) Second, that people may not have a grand theory of inductive reasoning does not render it suspect and/or to be dismissed. Our circumstances force us to recognise and respond to orderly and predictable patterns in the world: gravity and falls, food vs poison, dangerous plants, animals etc, potable vs undrinkable water, facts of reproduction, the pattern of day and night, land and sea, the dynamics of fire and much more. We therefore have to recignise and respect patterns. Deductive reasoning studies valid arguments wherein premises logically entail conclusions, but typically cannot establish start-points beyond doubt. Induction, by contrast takes in a big chunk of the rest: arguments where circumstances and premises support but do not strictly entail conclusions. Though, sometimes the conclusion may be sufficiently credible that it is practically certain. In both cases, we come back to needing intelligent and active, creative reasoners who can build warrant for credibly true beliefs. When they have done so to the point of moral certainty such that one would be irresponsible to dismiss the findings or otherwise act as though they are false, we have reason to accept the conclusions as knowledge in the soft sense. That brings us full circle to your attempt to redefine knowledge away from knowers. It fails. Inductive reasoning is responsible for most of humanity's knowledge base regarding an empirical world, the messy but vital world of credible but inevitably somewhat uncertain knowledge beyond the wider context of opinions and beliefs that do not have the degree of warrant as described. And, your dismissive opinion and attempted relabelling to the contrary notwithstanding, empirically grounded reasoning where circumstances and premises support conclusions does account for growth of knowledge. BTW, your attempt to turn Evolution into a researcher and even a Nobel Prize worthy Scientist fails as a case of the fallacy of turning an abstract descriptive term into an active creative force. Further to this, blind chance variations and mechanical necessity working through variation of populations and differential reproductive success etc has never been actually observed to have adequate causal capacity to originate body plan level novelties involving functionally specific, complex organisation and associated information beyond 500 - 1,000 bits. It remains a fact that the only actually known adequate cause of such FSCO/I is design. In short, without adequate warrant, you have appealed to a priori ideological imposition and have reified evolution into a grand designer. You are still dismissive of the point that there is a considerable body of evidence that we live in a unified, ordered partly intelligible world -- a cosmos, rather than a chaos. Indeed, the surfacing of this chaos thought suggests to me that the all too common and spreading, angrily motivated dismissal of the Judaeo-Christian ethical theistic worldview in our time is threatening to bring back the chaos vision that was a roadblock to the rise of science as a major cultural institution in other civilisations. But then, there is mounting evidence of a civilisation in decline, with a delusional death-wish leading to marches of folly all around. Now of course, you would suggest you are attempting to use induction to prove induction -- popular since Hume. Not so, I am instead working at world-foundational, comparative difficulties level, as is Avi Sion, whom I cited with approval. That is, we have the alternatives, chaos or cosmos at world-roots level to be evaluated on comparative difficulties across factual adequacy, coherence, explanatory power. The immediate point Sion made that the inference of disunity leads to implying a unity is indicative of the incoherence of the chaos option. In addition as I added, the very fact of finding that some patterns are non-uniform is itself an inductive generalisation that has no prospect of being overturned. So, there are good grounds for accepting evidence of uniformities with room for acknowledging limits or breakdowns when evidence of such is forthcoming. Further, the best current explanation approach readily accommodates such. Then, it is patent that the induction from the failure of some generalisation to the despairing of seeing order in the world patently lacks explanatory balance and power relative to the fallible best findings to date approach. The worldview level choice practically makes itself, in favour of cosmos not chaos. I will note, we have obviously gone to the deeper, worldview-foundation level, which is pre-theoretical but exerts a subtle influence on all that is built on it. In that context there is a spiral of observation, sensing and perceiving possible patterns, articulation into explanatory models including laws and theories etc, and further testing in light of experimental and observational studies. In such each facet influences the other and there is a dynamic, hopefully progressive, spiralling collaborative and social process which has been institutionalised in the sciences. In short, the picture of science and its progress you have painted is simplistic. A more realistic modelling process would be multi-faceted, interactive and would reflect a broad-based inherent complexity. I see you revert to the all too commonly seen polarising appeal to prejudice driven strawman dichotomy, natural vs supernatural. From the same passage in the laws Bk X on, there is another contrast which is more empirically relevant: the natural vs the ART-ificial. That is, that which credibly spontaneously arises from blind chance and mechanical necessity acting on initial circumstances in a dynamic-stochastic process, and that which comes about by creative art that manifests itself in artifacts typically exhibiting signs of design. The intelligent design scientific project, FYI and FTR, infers provisionally and inductively on empirically tested reliable observed sign to design as key causal factor. It does not infer to a designer as such, nor to the ontological nature of that designer. It is the case that on the world of life, from the birth of the modern design movement in the 1980's on, it has been explicitly and repeatedly acknowledged that inference on signs in the world of life do not in themselves ground claims to such design originating in designers being within or beyond the cosmos. Ironically, the projection in the teeth of facts to the contrary reflects the use of evolutionary thought over the past 150 years to try to put God out of a job. In short, it is a pale reflection of an uneasy conscience. I suggest that those who do such projections would be well advised to consult the work of Wallace, especially his The World of Life. Of course, there is another domain of design theory, the cosmological. Ever since Hubble's red shift observations from the 1920's spoke to the suggestion of expansion implicit in Relativistic cosmological models, the issue of a credible origin of our observed cosmos -- the only actually observed cosmos -- has been on the table. That unavoidably raises the issue that a beginning points to contingency of being and dependence on enabling causal factors external to such an entity. For, patently, non-being can have no causal powers. From the 1960's the background 2.7 K radiation observations has effectively taken the steady state model off the table as a serious contender. Multiply that by the growing pattern of findings that indicates astonishingly co-ordinated fine tuning, and we are led to the challenge that design is a reasonable candidate explanation for a cosmos with the kind of C-chemistry, aqueous medium terrestrial planet life-friendly physics we see. That indubitably raises the serious question of an extra-cosmic designer. And, the speculative metaphysics of increasingly exotic multiverse models, is just as much deeply philosophical in character. Which brings to bear many other worldview level issues such as the ontological-moral-mind issues that then lead to a cumulative case for ethical theism. Deep philosophy doubtless, but absolutely reasonable and subject to comparative difficulties assessment across factual adequacy, coherence and explanatory power. So, again, you are invited to do some rethinking. KFkairosfocus
July 17, 2015
July
07
Jul
17
17
2015
01:18 AM
1
01
18
AM
PDT
KF:
Evolutionary materialism is primarily a philosophical and ideological view, not a science, never mind the lab coat it currently wears.
The question of what is or is not science falls under the field of philosophy. Namely, the philosophy of science, right? Furthermore, no one has formulated a working theory of induction. Merely calling conjecture and criticism "inductivism" only muddies the waters. But, again, this isn't necessarily an issue. The problem is when you try to take induction seriously as an explanation for how knowledge grows. Your objections are an example of doing just that. And if your view has nothing to do with justificationism, then what other objections do you have against materialism? On the other hand, evolutionary theory does fit our best explanation of science. In fact, the two fall under the same universal explanation for the growth of knowledge. In science, we start out with a problem, which we then conjecture explanatory theory about how the world works to solve it. Then we use various ways to criticize that theory, which includes empirical tests, and discard errors we find. Evolution is a variation of conjecture, in the form of variations that are random to any problem to solve, and criticism, in the form of natural selection. Objections to this theory are inductive in nature because it assumes the unseen resembles the seen. The explanation for biological complexity is like an existing experience, such as a factory or a design studio. Neo-darwinism it's not an option because it doesn't resemble the seen. The universe was created by God, which is just an infinitely better version of human beings. Again, the assumption is that unseen resembles the seen. Empiricism was important step, because it helped promote empirical observations in science. However, it got the role those observations play backwards. Theories are tested by observations, not derived from them. The supernatural is a bad explanations because it is easily varied. This criteria is applicable to non-supernatural explanations as well, so claims that I'm merely exhibiting prejudice are unfounded. You continue on to attempt to assign a probability to a theory. However, it's unclear how that is even possible given that you have no way of calculating it. An example of how probably is misused is when ID proponents claim neo-Darwinism is improbable as a theory based on research on protein evolution, such as the probability of protein C evolving from protein A or protein B. But that tells us nothing about the probability of Darwinism, the theory, vs some other theory, let alone what are basically explanation-less theories. Not to mention that it make assumptions, such as that protein's C functionality in nature was the only way it could be brought about, that it was a specific target intended to be hit, etc., which has nothing to do with the intra-thoery question of the probably of which of two specific proteins, A or B, protein C evolved from under evolutionary theory and biology as we know it today. Probability simply isn't valid in this case. Yet, ID proponents commonly trot it out as if it were. Again, you need a theory before you can calculate the probably of anything. This is because theory determines what all the possible outcomes are before hand. So, reason always comes first. KF:
PS: Relabelling the normal process of empirical testing of inductive inferences and its provisionality/defeasibility then using that to suggest that inductive reasoning has not been used is an error.
First, I'm not merely relabeling it. That's a straw man. I'm suggesting something significantly different it happening in reality. Did Einstein merely relabel Newton's laws of motion? No, he too suggested something significantly different was happening in reality. Yet, for the most part, what we experience in our daily lives has not changed. That's possible because the unseen doesn't resemble the seen. Second, it's an error because.....? KF:
Likewise, the general experience of humanity of a predictable, stable common world in which like causes like, is already excellent pre-theoretic, worldview and commonsense grounds to accept that we live in a cosmos not a chaos; this is not warping perception of reality or begging questions, it is recognising that there are uniformities, regularities and order in the world so one should not be overly suspicious on encountering a fresh case though reasonable caution is also advisable because apparent patters with exceptions or overturning evidence are also a part of general experience.
Again, I've addressed this already.
Science explains the seen via the unseen. The unseen can be uniform in ways that are not revealed though the senses. For example, our explanation for how stars work indicates our sun will remain on it’s main sequence for another 5.7 billion years. However, if we had a different explanation about how stars work that indicated our sun had a lifespan of 4.6 billion years and would just suddenly extinguish itself [when it dies], we would wouldn’t expect it to rise tomorrow, despite the fact that it had been observed rising every morning for as long as human beings have existed.
Does this suggest the world is not uniform? No it doesn't. It assumes the unseen aspects of the world that we use to explain the seen are uniform. Your problem is that you're appealing to induction in subtle ways in assuming we can use it to make predictions that are true or probably true. This can take the form of “The future will resemble the past’, ‘the distant resembles the near,’ ‘the unseen resembles the seen’ and so on. But it doesn't. What was the clinching evidence that space-time is curved? It was a photograph, not of space-time, but of an eclipse, with a dot there rather than there.
I observe, we find nowhere so far good reason to abandon the view that knowledge (soft sense) is best viewed as warranted, credibly true belief, with the warrant being to some degree of moral certainty such that one would be irresponsible to treat a claim K that meets such a test as though NOT-K instead were to be presumed true.
You observe? In your experience? But isn't that exactly the point of contention? Again, that lets the naive version of induction off far to easily, as it accepts a number of misconceptions, which I've pointed out above. KF:
If arsenic is a known poison based on investigation and warrant, one would be ill advised indeed to treat it instead as food, or to disregard tests that show its significant presence in the body of an apparent murder victim.
I don't know why you keep ignoring key differences I've pointed out. Again, We start out with a problem, conjecture a theory about how the world works to solve it, then test that theory and discard errors we find. So, the problem might be, people or livestock are are getting sick when they visit a particular area or exposed to specific source of food or water. You then conjecture a theory about what factors are the cause. You might guess that arsenic, which was preset, has something to do with it, then decide to start testing that idea by empirical observation and tests. But nothing in our experience is the genesis of that theory. Observations may limit our possible theories, but they start out as a guess. And, even then, we look for explanatory reasons as to why arsenic is toxic in a chemical sense, etc., which leads to theories about us about safe limits, etc.Popperian
July 14, 2015
July
07
Jul
14
14
2015
06:27 AM
6
06
27
AM
PDT
PS: Relabelling the normal process of empirical testing of inductive inferences and its provisionality/defeasibility then using that to suggest that inductive reasoning has not been used is an error. PPS: Likewise, the general experience of humanity of a predictable, stable common world in which like causes like, is already excellent pre-theoretic, worldview and commonsense grounds to accept that we live in a cosmos not a chaos; this is not warping perception of reality or begging questions, it is recognising that there are uniformities, regularities and order in the world so one should not be overly suspicious on encountering a fresh case though reasonable caution is also advisable because apparent patters with exceptions or overturning evidence are also a part of general experience. Being reasonable and critically aware does not overthrow inductive reasoning but it holds it in balance. That is, on balance there is good commonsense reason to accept that if a stable pattern is observed that should be taken seriously until and unless circumstances emerge to show its limitations. That is, there is good reason to infer to the existence of a coherent pattern of lawlike regularities as a major feature of our world, a pattern amenable to detection and characterisation by observation, inductive inferences and experiment that substantiates the credible reliability of such summarised patterns as F = m*a or E = m*c^2 etc. Yes, this fits well with the Judaeo-Christian worldview and that is historically important in the founding era of modern science. In short the notion that an inductive approach cannot or does not adequately address development or growth of knowledge is historically falsified and indeed is patently ill-informed. Newton's summary in Opticks Query 31 and the broader view on inference to best current explanation provide a framework -- already cited and outlined (but not cogently responded to) -- for addressing this diversion. PPPS: I observe, we find nowhere so far good reason to abandon the view that knowledge (soft sense) is best viewed as warranted, credibly true belief, with the warrant being to some degree of moral certainty such that one would be irresponsible to treat a claim K that meets such a test as though NOT-K instead were to be presumed true. Truth, saying of what is that it is and of what is not that it is not. I later add: belief, implying accepting or acknowledging as true. If arsenic is a known poison based on investigation and warrant, one would be ill advised indeed to treat it instead as food, or to disregard tests that show its significant presence in the body of an apparent murder victim. Where, such knowledge is deeply embedded with inductive reasoning, in turn rooted in the general experience and rationality of humanity. The diversion on critical rationalism fails.kairosfocus
July 13, 2015
July
07
Jul
13
13
2015
01:56 AM
1
01
56
AM
PDT
Popperian Your "rich" comment is an ill advised ad hominem. Evolutionary materialism is primarily a philosophical and ideological view, not a science, never mind the lab coat it currently wears. For instance, observe Lewontin's key admission in his 1997 NYRB article:
the problem is to get them [= hoi polloi] to reject irrational and supernatural explanations of the world, the demons that exist only in their imaginations, and to accept a social and intellectual apparatus, Science, as the only begetter of truth [--> NB: this is a knowledge claim about knowledge and its possible sources, i.e. it is a claim in philosophy not science; it is thus self-refuting]. . . . To Sagan, as to all but a few other scientists, it is self-evident [--> actually, science and its knowledge claims are plainly not immediately and necessarily true on pain of absurdity, to one who understands them; this is another logical error, begging the question , confused for real self-evidence; whereby a claim shows itself not just true but true on pain of patent absurdity if one tries to deny it . . ] that the practices of science provide the surest method of putting us in contact with physical reality, and that, in contrast, the demon-haunted world rests on a set of beliefs and behaviors that fail every reasonable test [--> i.e. an assertion that tellingly reveals a hostile mindset, not a warranted claim] . . . . It is not that the methods and institutions of science somehow compel us to accept a material explanation of the phenomenal world, but, on the contrary, that we are forced by our a priori adherence to material causes [[--> another major begging of the question . . . ] to create an apparatus of investigation and a set of concepts that produce material explanations, no matter how counter-intuitive, no matter how mystifying to the uninitiated. Moreover, that materialism is absolute [[--> i.e. here we see the fallacious, indoctrinated, ideological, closed mind . . . ], for we cannot allow a Divine Foot in the door. {Billions and billions of demons. In case you imagine this citation to be "quote-mined" (another favourite ad hom laced dismissive argument) kindly cf the fuller annotated cite here.]
Boiled down, no evidence or reasoning that could challenge this ideological a priori is to be admitted. Fallacy of closed, ideological mind securing against effective empirical testing or possibility of falsification. (Which should ring a bell or two.) Even while, the pivotal scientific claims, on causal adequacy of blind chance variations and differential reproductive success of resulting varieties in ecological niches to create novel body plan level functional organisation and associated information beyond 500 - 1,000 bits has never been actually observed. Failing therefore, the vera causa (demonstrated, adequate cause) test. Further, analysis of the needle in haystack search for such config spaces shows that solar system or observed cosmos atomic and temporal resources are grossly inadequate to search out more than 1 straw to a cubical haystack comparably thick as our galaxy at the low end. This substantiates why the observation has been so elusive. There is therefore no proper comparison between proposing on evidence of trillions of actually observed cases that the best current, empirically anchored explanation of such FSCO/I is design, and the attempt before us to dismiss inductive reasoning as dubious. Most of our real world reasoning is at least in key parts inductive. And, science stands on the shoulders of logic, not the other way around. KFkairosfocus
July 13, 2015
July
07
Jul
13
13
2015
01:40 AM
1
01
40
AM
PDT
KF:
you have committed yourself to a view that would write off a major branch of logic.
First, isn't this rich coming from someone who has committed themselves to a view that would write off the most fundamental branch of biology? Second, I'm not suggesting there is no knowledge, or that it does not grow. In fact, I've said it is generally not a problem for people confuse or call conjecture and criticism "induction" as long as no one takes the idea seriously as an explanation for how knowledge grows or where theories come from. This is precisely what's happening here To use an example, Einstein better explained the motions of objects without completely causing us to rebuild bridges and buildings. This is despite suggesting something completely different was happening in reality. Not to mention that Einstein's theory unified notions of space and time with electromagnetism. I was going to say, "in a similar sense, Popper's better explanation and unification does not require us to rebuild buildings or bridges.", but it literally is the same sense. We do not build bridges based on experience, but based on explanations about how structures and materials work together. Conjecture and criticism is just a better explanation of what's happening there, in reality. KF:
The first comment is that, for good reason as outlined, there is no cause to dismiss inductive reasoning as suspect.
There are causes. And I've outlined them. Merely quoting definitions about what induction is does not address those issues. Yes, you can choose to call conjecture and criticism induction, but that just confuses the issue.
You seem to be concerned over how scientific theories come about, likely reflecting the scientism of our day.
You're conclusion is wrong. I'm not an empiricist, remember? Why do you keep responding if I am? I'm concerned about it because it's a key point of differentiation and is reflected in arguments made here on UD. We do not extrapolate theories from observations. Rather, theories start out as guesswork, conjectures about how the world works, which we then test and discard errors. Nothing you've quoted address this or indicates what principle would make the contrary possible.
My thought is broader, we need to address the worldviews level at one end and trivialities such as how we know food and drink as opposed to poison are good for us at the other.
Actually, my though is broader and deeper, because it goes beyond human knowledge and is concerned with how we even determine if an experience represents a repetition. To use an example, sit outside in the early morning with a mirror and video camera facing east. Look at the eastern sky. What do you experience? The sun rising. Look into the mirror. What do you experience? The sun rising. Look into the camera's LCD preview display while it's recording. What do you experience? The sun rising. Later in the day, play back the recording on your TV. What do you experience? The sun rising. Do we consider all of those experiences as the sun having risen four times in 24 hours,? No, we do not. That's because what experiences are considered repetition is based on explanatory theories, such as optics, geometry, photons, video recording, etc. And the same can be said about a cloudy morning. Those same theories tell us that sun is rising behind the clouds, rather than representing instance of the sun not rising at all, even if we do not experience it. Reason always has its way first.Popperian
July 12, 2015
July
07
Jul
12
12
2015
05:13 PM
5
05
13
PM
PDT
Popperian, you have committed yourself to a view that would write off a major branch of logic. That needs to be put on the table and addressed. I have taken time therefore to highlight that inductive reasoning continues to be pivotal in our rational life, even giving rise to whole branches of study such as statistics. In this general context, the position that needs very careful scrutiny is that which would lock out a whole province of reasoning to the point in the end that even the deliverances of our eyes and ears and our sense of the permanence of objects not currently in sight would become suspect. For, our experience and general knowledge of the world are of inductive character. I pause for a moment to make initial comments, DV to be followed up more specifically. The first comment is that, for good reason as outlined, there is no cause to dismiss inductive reasoning as suspect. As for its limitations etc, here is Newton, 300+ years past in Opticks, Query 31:
As in Mathematicks, so in Natural Philosophy, the Investigation of difficult Things by the Method of Analysis, ought ever to precede the Method of Composition. This Analysis consists in making Experiments and Observations, and in drawing general Conclusions from them by Induction, and admitting of no Objections against the Conclusions, but such as are taken from Experiments, or other certain Truths. For [speculative] Hypotheses are not to be regarded in experimental Philosophy. And although the arguing from Experiments and Observations by Induction be no Demonstration of general Conclusions; yet it is the best way of arguing which the Nature of Things admits of, and may be looked upon as so much the stronger, by how much the Induction is more general. And if no Exception occur from Phaenomena, the Conclusion may be pronounced generally. But if at any time afterwards any Exception shall occur from Experiments, it may then begin to be pronounced with such Exceptions as occur. By this way of Analysis we may proceed from Compounds to Ingredients, and from Motions to the Forces producing them; and in general, from Effects to their Causes, and from particular Causes to more general ones, till the Argument end in the most general. This is the Method of Analysis: And the Synthesis consists in assuming the Causes discover'd, and establish'd as Principles, and by them explaining the Phaenomena proceeding from them, and proving the Explanations.
With some updating of language and a recognition of the modern sense of induction, that is still pretty good. No wonder it is what probably lies behind most school level presentations of "the" scientific method. You seem to be concerned over how scientific theories come about, likely reflecting the scientism of our day. My thought is broader, we need to address the worldviews level at one end and trivialities such as how we know food and drink as opposed to poison are good for us at the other. I also point, in brief, that the inductive pattern known as inference to the best [current] explanation easily addresses the rise, support and fall of scientific theories and models more generally. When observed facts F1, F2, . . . Fn are there to be accounted for an explanation Ei that entails them is a candidate, which may then go on to predict P1 to Pm. Alternatives may diverge at crucial points PCj. The theory which performs best in the contest across {F, P and PC} is best current and under sufficient testing may prove empirically reliable. Thus, it becomes preferable, but is never to be equated with a fact as Ei => { F, P, PC } does not allow reversal of the implication. Implication is not equivalence. Thus scientific theories are more or less reliable explanatory models, not facts in themselves -- as was noted above. (BTW, that is one point where there is a serious blunder that is too often made on origins topics, where the degree of testing is significantly lower than for cases where we may directly inspect events or where we may carry out controlled experiments.) But obviously, a theory that covers F, is successful with P and is good on PC is well supported and may actually have a chance to be true. Just, theories are not known facts. Also, there are cases where Ei and Ej are empirically equivalent, and not just in a sense of grueness or the like. In such cases secondary and far more complex judgements on things such as elegance come into play. There are odd cases where it turns out that theories are in effect dual to each other once proper transformations are made. So, the development of scientific theories fits in with induction, it does not overturn it. KF PS: A nice short summary from a lecture course: https://www.msu.edu/~orourk51/130-Phil/Spring2015/Lectures/130S15-Lex11-StatisticalArgs.pdfkairosfocus
July 12, 2015
July
07
Jul
12
12
2015
11:07 AM
11
11
07
AM
PDT
KF, Let me approach this from another angle, because we do not seem to be making progress. For example, I've found nothing in #52 that actually address the points I made. Rather, what you seem to have done is merely look up induction, then quote definitions that do not reflect the naive variant of it. But, again this ignores criticism regarding its supposed role in the origin of developing theories. Then again, if I've missed it, please kindly quote from #52 and expand on how it's relevant. Nor am I saying that statistics are completely meaningless. I'm saying that the use of probability is grossly overestimated in selecting between theories. Nothing address this in #57 either. From #57 I wrote:
Induction shares the same flaw as Lamarckism and creationism. It assumes that new knowledge is somehow present at the outset in experience or can be derived from experience. That’s a key aspect of Induction I’m referring to. In those cases, the explanation for that new knowledge is either absent, irrational or supernatural, which is easy varied.
KF:
Such a dismissal is little more than a sez me, dogmatic appeal to prejudice — prejudices that do little more than show a secularist flag, raising the issue that you need to take seriously the self-referential incoherence and self-falsification of evolutionary materialist scientism as already addressed. Prejudice, in the face of a huge mass of well warranted fact of experience. But, notwithstanding your undue skepticism: we can and do draw on experience in developing knowledge as is massively evident all around.
Except, my criticism isn't mere prejudice. The criticism of being a bad explanations (easily varied) is not limited to the supernatural. Authoritative sources of knowledge is a particular philosophy, and a particularly bad one as well. Apparently, you jumped when you ran across the word "supernatural". That's a red herring. Being abstract and having no defined limitations, your designer is essentially an authoritative source of knowledge. No better explanation can be had other than, "That's just what some designer must have wanted". And that is by design. I wrote:
To say something is true or probably true, one must interpret observations in such a way to determine what it means for anything to actually represent a repetition. But that doesn’t come from experience. Theory has its way first. So, we cannot mechanically derive theories from observations. Rather, we conjecture theories, then criticize them. in the case of science, criticism takes the form of empirical observations. Note how criticism plays the same role in both science, philosophy and even biological complexity.
KF:
First, and crucially, observational data is not “criticism,” it is collected credible facts in a situation. Such may be used in testing or criticising a theory, model, proposed decision or policy etc but it is not in itself criticism per se. Indeed, once responsibly collected, analysed and reported is is a case in point of exactly the sort of soft sense knowledge I have described.
Yes, one can observe things all day long without the intent to use those observations in a critical way. This does not mean that observations via empirical tests do not play a critical role in science, as opposed to the idea that we supposedly derive theories from observations. Nor can observations merely be a collection of facts because those observations must first be interpreted though the lens of a theory. KF:
I must next ask, did you really interact with 52? In particular, with this from Avi Sion:
The problem is that you're suggesting the unseen, including any uniformities it might have, resembles the seen. But it doesn't. Science explains the seen via the unseen. The unseen can be uniform in ways that are not revealed though the senses. For example, our explanation for how stars work indicates our sun will remain on it's main sequence for another 5.7 billion years. However, if we had a different explanation about how stars work that indicated our sun had a lifespan of 4.6 billion years and would just suddenly extinguish itself, we would wouldn't expect it to rise tomorrow, despite the fact that it had been observed rising every morning for as long as human beings have existed. So, it would seem that you are either confused about my position or you are deliberately being obtuse, as you've failed to address the points I've raised. Until you do, let me point out that just being mistaken about how knowledge grows really isn't a problem. This is because one can merely choose to call conjecture and criticism "induction" and still make progress. The problem of induction can be summarized as: we need induction in some form to justify or found the existence of knowledge. However, no one has formed a principle of induction that actually works in practice. Yet, our experience indicates we use induction and are successful at employing it. So, if not the naive variant of induction, there must be some sophisticated / modern version, which we actually use, which we still call induction. So, we're stuck trying to solve the mystery of how something that is supposedly necessary and we supposedly experience doing can be done despite the lack of a working principle that explains it. IOW, part of the problem of induction includes the ideas that, without induction, there could be no knowledge and that we must use it because we experience using it successfully, over and over again. However, experience can be mistaken. This is an important point that spans the entire series of posts I've commented on. In this case, experience is mistaken about deriving theories from observations. It may seem that's what we do, but that idea does not survive rational criticism. But, again, that doesn't necessarily matter, per se, because one can be mistaken about how we make progress, yet still make progress. The problem comes into play when someone tries to actually take that mistake seriously as an explanation for the growth of knowledge. This includes many of the objections found here on UD. And I've pointed those out where appropriate. Specifically, the idea that we derive theories from observations, which appeared as objections is the previous two posts.Popperian
July 12, 2015
July
07
Jul
12
12
2015
07:39 AM
7
07
39
AM
PDT
Mung, I would suggest that there are senses in which we may assign probabilities to fact claims, and a broader sense in which we may regard things as more or less credible, in some cases amounting to moral certainty: e.g. at earth's surface gravity field strength is about 9.8 N/kg, or that the earth is round and orbits the Sun once per year. KFkairosfocus
July 11, 2015
July
07
Jul
11
11
2015
03:42 AM
3
03
42
AM
PDT
Popperian: I draw your attention to the citation from the well-known IEP in 52 above (and as well to the fairly well known discipline commonly known as statistics, as just one example):
A deductive argument is an argument that is [–> assuming honest discussion!] intended by the arguer to be (deductively) valid, that is, to provide a guarantee of the truth of the conclusion provided that the argument’s premises (assumptions) are true. This point can be expressed also by saying that, in a deductive argument, the premises are intended to provide such strong support for the conclusion that, if the premises are true, then it would be impossible for the conclusion to be false. An argument in which the premises do succeed in guaranteeing the conclusion is called a (deductively) valid argument. If a valid argument has true premises, then the argument is said to be sound . . . . An inductive argument is an argument that is intended by the arguer merely to establish or increase the probability of its conclusion. In an inductive argument, the premises are intended only to be so strong that, if they were true, then it would be unlikely that the conclusion is false. There is no standard term for a successful inductive argument. [–> I have often found cogency and sometimes rational convincing power, occasionally proof in a weaker sense esp. in older sources such as Greenleaf] But its success or strength is a matter of degree, unlike with deductive arguments. A deductive argument is valid or else invalid. The difference between the two kinds of arguments does not lie solely in the words used; it comes from the relationship the author or expositor of the argument takes there to be between the premises and the conclusion. If the author of the argument believes that the truth of the premises definitely establishes the truth of the conclusion (due to definition, logical entailment, logical structure, or mathematical necessity), then the argument is deductive. If the author of the argument does not think that the truth of the premises definitely establishes the truth of the conclusion, but nonetheless believes that their truth provides good reason to believe the conclusion true, then the argument is inductive.
In response, you seem to have ignored the crucial development implied in the definition presented and just reiterated your view that inductive reasoning is dead per impact of Popper et al. What would be fairer for you to have done, would have been to acknowledge that in recent times there has been a refinement of the understanding of the nature of induction, from inference to the general on particular cases (now one particular form) to defeasible, empirically based reasoning intended to establish the plausibility or credibility or likelihood of a conclusion. Which of course directly puts a world of probabilistic, statistical reasoning at the heart of the modern practice of inductive reasoning, as well as the directly linked world of scientific, engineering, medical + health, managerial and policy praxis. I cite the American Statistical Association, by way of illustration:
Statistics is the science of learning from data, and of measuring, controlling, and communicating uncertainty; and it thereby provides the navigation essential for controlling the course of scientific and societal advances (Davidian, M. and Louis, T. A., 10.1126/science.1218685). Statisticians apply statistical thinking and methods to a wide variety of scientific, social, and business endeavors in such areas as astronomy, biology, education, economics, engineering, genetics, marketing, medicine, psychology, public health, sports, among many. "The best thing about being a statistician is that you get to play in everyone else's backyard." (John Tukey, Bell Labs, Princeton University) Many economic, social, political, and military decisions cannot be made without statistical techniques, such as the design of experiments to gain federal approval of a newly manufactured drug.
This is strong evidence as to the significance of quantitative inductive reasoning in our day, and direct demonstration that a wide range of responsible praxis routinely works within the paradigm of inductive reasoning on matters of the highest import. Such should help you to reconsider. The more fundamental point is, we still confront an empirical world and are forced to address evident patterns of observation or experience, drawing responsible conclusions to guide praxis as to what is credibly true at a level of warrant sufficient that to treat such matters as false would be irresponsible. In short, we see the significance of:
1: truth, as that which says of what is that it is, and of what is not that it is not, and 2: knowledge (soft sense) as warranted, credibly true belief, where warrant addresses the material circumstances such that it attains some level of moral certainty.
It is fair comment for me to note that while one can find academics and adherents of all sorts of viewpoints, the above summarises effective and well grounded understanding and praxis in light of the material facts, issues and concerns. They may fairly be described as knowledge of what knowledge is, and not in a circular or incoherent sense. I note too that the criterion of moral certainty is in the end an issue of intellectual virtue in light of relevant duties of care, not primarily an exercise in probability or Bayesian reasoning in particular. Beyond a certain threshold we in our finitude and fallibility must decide and act; where, to postpone decision is itself a decision. We must act with consequences on the line. In good conscience, what OUGHT we to accept and in that light what OUGHT we to do. In short there are much broader worldview implications and responsibilities on the table than may be at once apparent in some ivory tower seminar room. Where also, inductive reasoning at a formal or informal level likely constitutes the basis of the bulk of our formal and informal knowledge regarding significant praxis. At this stage, it is opinions to the contrary that lack adequate warrant. This is illustrated by how you tried to dismissively cluster inductive reasoning with other allegedly suspect views:
Induction shares the same flaw as Lamarckism and creationism. It assumes that new knowledge is somehow present at the outset in experience or can be derived from experience. That’s a key aspect of Induction I’m referring to. In those cases, the explanation for that new knowledge is either absent, irrational or supernatural, which is easy varied.
Such a dismissal is little more than a sez me, dogmatic appeal to prejudice -- prejudices that do little more than show a secularist flag, raising the issue that you need to take seriously the self-referential incoherence and self-falsification of evolutionary materialist scientism as already addressed. Prejudice, in the face of a huge mass of well warranted fact of experience. But, notwithstanding your undue skepticism: we can and do draw on experience in developing knowledge as is massively evident all around. And BTW, if you are summarising inference to design as credible causal process on tested reliable observed signs in terms of "some abstract designer that has no limitations" you are erecting a gross strawman. You continue:
To say something is true or probably true, one must interpret observations in such a way to determine what it means for anything to actually represent a repetition. But that doesn’t come from experience. Theory has its way first. So, we cannot mechanically derive theories from observations. Rather, we conjecture theories, then criticize them. in the case of science, criticism takes the form of empirical observations. Note how criticism plays the same role in both science, philosophy and even biological complexity.
First, and crucially, observational data is not "criticism," it is collected credible facts in a situation. Such may be used in testing or criticising a theory, model, proposed decision or policy etc but it is not in itself criticism per se. Indeed, once responsibly collected, analysed and reported is is a case in point of exactly the sort of soft sense knowledge I have described. I must next ask, did you really interact with 52? In particular, with this from Avi Sion:
We might . . . ask – can there be a world without any ‘uniformities’? A world of universal difference, with no two things the same in any respect whatever is unthinkable. Why? Because to so characterize the world would itself be an appeal to uniformity. A uniformly non-uniform world is a contradiction in terms. Therefore, we must admit some uniformity to exist in the world. The world need not be uniform throughout, for the principle of uniformity to apply. It suffices that some uniformity occurs. Given this degree of uniformity, however small, we logically can and must talk about generalization and particularization. There happens to be some ‘uniformities’; therefore, we have to take them into consideration in our construction of knowledge. The principle of uniformity is thus not a wacky notion, as Hume seems to imply . . . . The uniformity principle is not a generalization of generalization; it is not a statement guilty of circularity, as some critics contend. So what is it? Simply this: when we come upon some uniformity in our experience or thought, we may readily assume that uniformity to continue onward until and unless we find some evidence or reason that sets a limit to it. Why? Because in such case the assumption of uniformity already has a basis, whereas the contrary assumption of difference has not or not yet been found to have any. The generalization has some justification; whereas the particularization has none at all, it is an arbitrary assertion. It cannot be argued that we may equally assume the contrary assumption (i.e. the proposed particularization) on the basis that in past events of induction other contrary assumptions have turned out to be true (i.e. for which experiences or reasons have indeed been adduced) – for the simple reason that such a generalization from diverse past inductions is formally excluded by the fact that we know of many cases [[of inferred generalisations; try: “we can make mistakes in inductive generalisation . . . “] that have not been found worthy of particularization to date . . . . If we follow such sober inductive logic, devoid of irrational acts, we can be confident to have the best available conclusions in the present context of knowledge. We generalize when the facts allow it, and particularize when the facts necessitate it. We do not particularize out of context, or generalize against the evidence or when this would give rise to contradictions . . .[[Logical and Spiritual Reflections, BK I Hume’s Problems with Induction, Ch 2 The principle of induction.]
Yes, we do see in a way shaped by our worldviews and our own sense of unified experience -- I note the over-dominance of science in your use of "theory" here, but such comes with diversity of individuals and good reason to expect at least partial uniformities based on things like: we find we need to breathe and eat and drink, that certain substances are required, that things tend to fall down, that certain objects regularly cycle in our sky -- including the sun, that there is a sky, an earth, there are bodies of a commonly experienced substance in several forms: water, and more. Despite views that may expect a chaos, there is a patent invitation to perceive regularities that are reasonably predictable. So the principle of uniformity is pre-theoretical, world-experience based and can be shown reasonably warranted on pain of absurdity. In short, we start from commonsense, or we do not start. Such may be refined and corrected based on study, but it must not be tossed aside dismissively, on pain of self-referential absurdity. And so forth. I suggest to you that you work your way through the rest of 52 above, which for example shows a reason why inductive generalisation is not a dubious project, and links on how a properly balanced worldviews approach on comparative difficulties addresses the issues of infinite regress and circularity leading to standing on finitely remote first plausibles that anchor a responsible, reasonable perspective or faith. Where also, self-evident plumbline truths allow us to test the quality of our thinking. Finally, I join with Mung: " if you have a theory of knowledge we would surely like to know what it consists of." KFkairosfocus
July 11, 2015
July
07
Jul
11
11
2015
03:26 AM
3
03
26
AM
PDT
Popperian, if you have a theory of knowledge we would surely like to know what it consists of. There is no such thing as "probably true." That's just silly talk. Nothing survives criticism, which is why it is irrational to be a "critical rationalist."Mung
July 10, 2015
July
07
Jul
10
10
2015
06:39 PM
6
06
39
PM
PDT
KF;
For instance, induction has hardly been abandoned as a reasonable and indeed main branch of logic. Indeed, in the modern sense such arguments are defeasible arguments that render a conclusion more plausible or likely on premises that are typically observational.
But there must be more than an idea merely not having been abandoned as reasonable and existing as a main branch in its field. If that were the case, then it would long have been time for Uncommon Descent to pack it in and turn off the lights: Neo-Darwinism has hardly been abandoned as a reasonable explanation for the growth of biological complexity and is a main branch in biology. This is a red herring to the issue that spanned each of the threads I commented on. Induction shares the same flaw as Lamarckism and creationism. It assumes that new knowledge is somehow present at the outset in experience or can be derived from experience. That's a key aspect of Induction I'm referring to. In those cases, the explanation for that new knowledge is either absent, irrational or supernatural, which is easy varied. To say something is true or probably true, one must interpret observations in such a way to determine what it means for anything to actually represent a repetition. But that doesn't come from experience. Theory has its way first. So, we cannot mechanically derive theories from observations. Rather, we conjecture theories, then criticize them. in the case of science, criticism takes the form of empirical observations. Note how criticism plays the same role in both science, philosophy and even biological complexity. The kind of criticism changes, not the role it plays. This is why I describe Popper's contribution as a theory for the universal growth of knowledge. When you step back look at the big picture, the uniting thread is variation and criticism of some form or another. For example, you wrote.
Obviously, machines may embed knowledge or acquire data by tested reliable means which are functional knowledge, but in every actually observed case of such, they are contrivances tracing to the action of responsibly free and rational intelligent agents.
However, "every observed case" in the above is only a continuation in the context of a specific theory by which to interpret observations. Reason always has its way first. Yet, you seem to be appealing to observations as if they can be extrapolated to form the theory, make it true or probably true. This simply does not survive criticism. Probably simply is the wrong category in which to select between theories because probability is only valid in the content of theory itself - which determines what the possible outcomes are. So, without the theory first, you can't calculate it. Nor can you calculate the probably of un-conceived theories for the same phenomena to compare them with. The same can be said for explanation-less theories. The point I've been trying to present, at a top level, is that false explanations for the growth of biological complexity can be traced to the underlying bad philosophy regarding the growth of knowledge which it is based on. This includes the current crop of ID (some abstract designer that has no limitations) and creationism. Again, the explanation for that knowledge is either absent, irrational or supernatural, which is easy varied.Popperian
July 10, 2015
July
07
Jul
10
10
2015
04:32 PM
4
04
32
PM
PDT
testkairosfocus
July 10, 2015
July
07
Jul
10
10
2015
07:24 AM
7
07
24
AM
PDT
F/N: More from Avi Sion responding to Hume's doubts: >>Hume doubted the validity of generalization because he thought that we adopt a general proposition like All X are Y, only on the basis of the corresponding particular Some X are Y. But if the latter was sufficient to (inductively) establish the former, then when we were faced with a contingency like Some X are Y and some X are not Y, we would be allowed to generalize both the positive and negative particulars, and we would find ourselves with a contradiction[2] in our knowledge, viz. with both All X are Y and No X are Y. But since contradiction is error, according to the 2nd law of thought, it follows that a particular is not by itself enough to confirm a generality. To do so, we need also to first adduce that the opposite particular is not currently justified. Note well what we have shown here: this criterion for generalization follows from the law of non-contradiction. Hume and his skeptical successors did not take this additional criterion into account. They noticed the aspect of ‘confirmation’, but ignored that of ‘non-rejection’. The uniformity principle ought to be viewed as an application of a much larger and important principle, which we may simply call the principle of induction (in opposition to the so-called problem of induction). This all-important principle could be formulated as follows: given any appearance, we may take it to be real, until and unless it is found to be illusory.[3] This is the fundamental principle of inductive logic, from which all others derive both their form and their content. And indeed, this is the way all human beings function in practice (with the rare exception of some people, like Hume, who want to seem cleverer than their peers). It is, together with Aristotle’s three laws of thought, the supreme principle of methodology, for both ordinary and scientific thought, whatever the domain under investigation[4]. Indeed, we could construe this principle of induction as the fourth law of thought. Just as the three laws proposed by Aristotle are really three facets of one and the same law, so also this fourth law should be viewed as implicit in the other three. Induction being the most pragmatic aspect of logic, this principle is the most practical of the foundations of rational discourse. The principle of induction is a phenomenological truth, because it does not presume at the outset that the givens of appearance are real or illusory, material or mental, full or empty, or what have you. It is a perfectly neutral principle, without prejudice as to the eventual content of experience and rational knowledge. It is not a particular worldview, not an a priori assumption of content for knowledge. However, in a second phase, upon reflection, the same principle favors the option of reality over that of illusion as a working hypothesis. This inbuilt bias is not only useful, but moreover (and that is very important for skeptics to realize) logically rock solid, as the following reasoning clearly shows: This principle is self-evident, because its denial is self-contradictory. If someone says that all appearance is illusory, i.e. not real, which means that all our alleged knowledge is false, and not true, that person is laying claim to some knowledge of reality (viz. the knowledge that all is unreal, unknowable) – and thus contradicting himself. It follows that we can only be consistent by admitting that we are indeed capable of knowing some things (which does not mean everything). It follows that the initial logical neutrality of appearance must be reinterpreted as in all cases an initial reality that may be demoted to the status of illusion if (and only if) specific reasons justify it. Reality is the default characterization, which is sometimes found illusory. Knowledge is essentially realistic, though in exceptional cases it is found to be unrealistic. Such occasional discoveries of error are also knowledge, note well; they are not over and above it. If we did not adopt this position, that appearance is biased towards reality rather than illusion, we would be stuck in an inextricable agnosticism. Everything would be “maybe real, maybe illusory” without a way out. But such a problematic posture is itself a claim of knowledge, just like the claim that all is illusory, and so self-inconsistent too. It follows that the interpretation of appearance as reality until and unless otherwise proved is the only plausible alternative.[5] If appearance were not, ab initio at least, admitted as reality rather than as illusion or as problematic, we would be denying it or putting it in doubt without cause – and yet we would be granting this causeless denial or doubt the status of a primary truth that does not need to be justified. This would be an arbitrary and self-contradictory posture – an imposture posing as logical insight. All discourse must begin with some granted truth – and in that case, the most credible and consistent truth is the assumption of appearance as reality unless or until otherwise proved.>> Time for some fresh thinking. KFkairosfocus
July 10, 2015
July
07
Jul
10
10
2015
06:12 AM
6
06
12
AM
PDT
Popperian, There is a clear impasse. Instead of a fruitless points-counterpoints rhetorical exchange, I think some things need to be put on the table. Whether or no you find the following acceptable, it will help onlookers, current and future. Now, Popper et al have made significant contributions to epistemology particularly tied to the tides of science across the past century, but their insights are not the only ones, nor would it be fair to suggest such have either the first or the last word much less the only word worth listening to. For instance, induction has hardly been abandoned as a reasonable and indeed main branch of logic. Indeed, in the modern sense such arguments are defeasible arguments that render a conclusion more plausible or likely on premises that are typically observational. Where, a major approach to such reasoning is inference to the best [current] explanation. As IEP summarises:
A deductive argument is an argument that is [--> assuming honest discussion!] intended by the arguer to be (deductively) valid, that is, to provide a guarantee of the truth of the conclusion provided that the argument's premises (assumptions) are true. This point can be expressed also by saying that, in a deductive argument, the premises are intended to provide such strong support for the conclusion that, if the premises are true, then it would be impossible for the conclusion to be false. An argument in which the premises do succeed in guaranteeing the conclusion is called a (deductively) valid argument. If a valid argument has true premises, then the argument is said to be sound . . . . An inductive argument is an argument that is intended by the arguer merely to establish or increase the probability of its conclusion. In an inductive argument, the premises are intended only to be so strong that, if they were true, then it would be unlikely that the conclusion is false. There is no standard term for a successful inductive argument. [--> I have often found cogency and sometimes rational convincing power, occasionally proof in a weaker sense esp. in older sources such as Greenleaf] But its success or strength is a matter of degree, unlike with deductive arguments. A deductive argument is valid or else invalid. The difference between the two kinds of arguments does not lie solely in the words used; it comes from the relationship the author or expositor of the argument takes there to be between the premises and the conclusion. If the author of the argument believes that the truth of the premises definitely establishes the truth of the conclusion (due to definition, logical entailment, logical structure, or mathematical necessity), then the argument is deductive. If the author of the argument does not think that the truth of the premises definitely establishes the truth of the conclusion, but nonetheless believes that their truth provides good reason to believe the conclusion true, then the argument is inductive.
In short, it is hardly well established that inductive reasoning is dubious or even swept away; not least in an era dominated by statistics and linked considerations. Nor, would it be fair to imagine that it cannot increase knowledge or credibly access truth. Indeed, in an empirical world, probably most arguments are inductive. So, no, for good reason I do not see Hume's strictures or those of his successors to date as decisive and fatal to the project of induction. Avi Sion provides a useful perspective on the inference to general patterns, i.e. the principle of uniformity:
We might . . . ask – can there be a world without any ‘uniformities’? A world of universal difference, with no two things the same in any respect whatever is unthinkable. Why? Because to so characterize the world would itself be an appeal to uniformity. A uniformly non-uniform world is a contradiction in terms. Therefore, we must admit some uniformity to exist in the world. The world need not be uniform throughout, for the principle of uniformity to apply. It suffices that some uniformity occurs. Given this degree of uniformity, however small, we logically can and must talk about generalization and particularization. There happens to be some ‘uniformities’; therefore, we have to take them into consideration in our construction of knowledge. The principle of uniformity is thus not a wacky notion, as Hume seems to imply . . . . The uniformity principle is not a generalization of generalization; it is not a statement guilty of circularity, as some critics contend. So what is it? Simply this: when we come upon some uniformity in our experience or thought, we may readily assume that uniformity to continue onward until and unless we find some evidence or reason that sets a limit to it. Why? Because in such case the assumption of uniformity already has a basis, whereas the contrary assumption of difference has not or not yet been found to have any. The generalization has some justification; whereas the particularization has none at all, it is an arbitrary assertion. It cannot be argued that we may equally assume the contrary assumption (i.e. the proposed particularization) on the basis that in past events of induction other contrary assumptions have turned out to be true (i.e. for which experiences or reasons have indeed been adduced) – for the simple reason that such a generalization from diverse past inductions is formally excluded by the fact that we know of many cases [[of inferred generalisations; try: "we can make mistakes in inductive generalisation . . . "] that have not been found worthy of particularization to date . . . . If we follow such sober inductive logic, devoid of irrational acts, we can be confident to have the best available conclusions in the present context of knowledge. We generalize when the facts allow it, and particularize when the facts necessitate it. We do not particularize out of context, or generalize against the evidence or when this would give rise to contradictions . . .[[Logical and Spiritual Reflections, BK I Hume's Problems with Induction, Ch 2 The principle of induction.]
Likewise, a worldviews approach on comparative difficulties reasonably addresses the problem of infinite regress of warrant or circularity. As Avi Sion just illustrated, comparative difficulties . . . factual adequacy, coherence, balanced explanatory power . . . across live option "serious" alternatives allows us to choose a reasonable best current position that rests on finitely remote first plausibles and key plumbline self-evident truths that ground a view as a reasonable faith, cf. here on. In such a context it makes excellent sense to view truth as accurate description of reality, a goal and challenge: to say of what is that it is, and of what is not that it is not. Then, we may see knowledge as warranted, credibly true belief where warrant here denotes a degree of moral certainty such that to be aware of the warrant but act as though the warranted claim were false would be irresponsible. Of course in limited key cases truths may be known to absolute certainty. In other cases, we have views, opinions, claims, beliefs etc that are not knowledge or that while being known to one or a few, are not generally understood to be known. I would suggest that this is a reasonable, balanced and satisfactory, effective position that needs not unduly defer to skeptical or hyperskeptical objections. Nor is it a novelty, though it obviously reflects developments of recent decades. I again draw attention to Simon Greenleaf as already cited from Bk I Ch 1 of his Treatise on Evidence, summing up the responsible, informed view of the legal profession c 1850 based on centuries of reflective, philosophically informed praxis:
Evidence, in legal acceptation, includes all the means by which any alleged matter of fact, the truth of which is submitted to investigation, is established or disproved . . . None but mathematical truth is susceptible of that high degree of evidence, called demonstration, which excludes all possibility of error [–> Greenleaf wrote almost 100 years before Godel], and which, therefore, may reasonably be required in support of every mathematical deduction. Matters of fact are proved by moral evidence alone; by which is meant, not only that kind of evidence which is employed on subjects connected with moral conduct, but all the evidence which is not obtained either from intuition, or from demonstration. In the ordinary affairs of life, we do not require demonstrative evidence, because it is not consistent with the nature of the subject, and to insist upon it would be unreasonable and absurd. The most that can be affirmed of such things, is, that there is no reasonable doubt concerning them. The true question, therefore, in trials of fact, is not whether it is possible that the testimony may be false, but, whether there is sufficient probability of its truth; that is, whether the facts are shown by competent and satisfactory evidence. Things established by competent and satisfactory evidence are said to be proved. By competent evidence, is meant that which the very-nature of the thing to be proved requires, as the fit and appropriate proof in the particular case, such as the production of a writing, where its contents are the subject of inquiry. By satisfactory evidence, which is sometimes called sufficient evidence, is intended that amount of proof, which ordinarily satisfies an unprejudiced mind, beyond reasonable doubt. The circumstances which will amount to this degree of proof can never be previously defined; the only legal test of which they are susceptible, is their sufficiency to satisfy the mind and conscience of a common man; and so to convince him, that he would venture to act upon that conviction, in matters of the highest concern and importance to his own interest. [A Treatise on Evidence, Vol I, 11th edn. (Boston: Little, Brown, 1888) ch 1., sections 1 and 2. Shorter paragraphs added. (NB: Greenleaf was a founder of the modern Harvard Law School and is regarded as a founding father of the modern Anglophone school of thought on evidence, in large part on the strength of this classic work.)]
Obviously, machines may embed knowledge or acquire data by tested reliable means which are functional knowledge, but in every actually observed case of such, they are contrivances tracing to the action of responsibly free and rational intelligent agents. Likewise, animals etc know through being programmed to take in and process reasonably reliable data in reliable ways, though obviously their knowledge is imperfect and limited even more than our own. That elephants never forget is notorious. Dogs recognise master's car coming around the corner and get ready to greet an obviously known master. Chickens, agoutis and iguanas by and large by instinct know to avoid cars and hunters or potential hunters. But the very word suggests implanted. And of course the requisite far more complex, specifically functional organisation and information here involved, points to a similar source in the wisdom of a designer. Never mind speculative theories to the contrary, until such pass the vera causa test of actual observed origin of FSCO/I by blind chance and mechanical necessity they decisively lack warrant. So, there is a reasonable understanding of knowledge and of truth on the table, together with a framework that shows that there is no reason to give up inductive reasoning as dubious. KF PS: I have already pointed out i/l/o the Gettier issue, why warrant has been distinguished from justification. The point that at the threshold there is some level of moral certainty would imply that there is a degree of objectivity involving reasonably accessible material factors such that say the blind luck of chancing on a generally reliable clock tower at 11:56 am where it was stuck at 11:56 pm, and the like, are addressed. (Such also draws out why generally speaking we want scientific models to avoid being ad hoc and to make broad-ranging specific predictions as such will more readily expose limitations, but no finite number of tests can ever force a particular conclusion; theories are best explanations to date or may be contenders for such.) And the infinite regress or circularity issues were long since addressed through a comparative difficulties worldviews approach as outlined. PPS: As citations from Rosenberg and others readily show, lab coat clad evolutionary materialism is a patent major issue, and it is also multiply self-referentially incoherent. PPPS: Kindly note 16 above:
16 kairosfocus July 7, 2015 at 1:59 pm Mapou, Start with the classical view: justified, true belief. Add, truth is what says of what is that it is and of what is not that it is not. Belief, here, means some relevant responsibly free intelligent subject accepts something as true. But that may not be reasonably warranted, even if true. It is possible to believe a true thing without adequate warrant. Mix in Gettier and the softening where we speak of things that are warranted to some degree of moral certainty but are open to the possibility of error. As is so for most real world knowledge. We then see that justification (as in I am personally justified . . . ) broadens to warrant in principle accessible to others. Warrant to some responsible degree of certainty [= some degree of moral certainty] then makes what one is inclined to accept as true to be credible . . . reasonable, reliable or trustworthy, worth acknowledging. So, one has good grounds to accept as true though not necessarily absolute certainty, grounds that are sufficiently broad to be beyond the Gettier issue of internally justified but on a wider view not warranted. One actually does live up to responsibility and accepts as true. One then knows in the relevant soft-form sense: warranted, credibly true belief.
kairosfocus
July 10, 2015
July
07
Jul
10
10
2015
01:24 AM
1
01
24
AM
PDT
KF:
Popperian, it is beginning to look like Lincoln’s riddle: “if we call the tail of a sheep a leg how many legs does a sheep have?” The correct answer is still four, twisting words does not change realities, never mind 2 + 2 = whatever The Party needs it to be, Mr Smith.
Again, I wrote:
As for my definition, progress can often take the form of unification. That is, seemly unrelated phenomena are explained by new or common theory that is generalized. This is what I’m suggesting Popper did in the field of epistemology; develop a universal theory for the growth of knowledge.
So, no. I'm not merely trying to call a sheep tail a leg for no reason at all, as you seem to be trying to suggest. I would again ask, what is your explanation for the universal growth of knowledge, but you seem opposed to the very idea of the existence of such a thing.
KF: Dismissing knowledge and truth then pushing in novelties into those terms does not change the underlying context where such are relevant and important realities; this is not some pomo 101 where if I don’t conform, my grades suffer.
Except, that's not what I'm doing. Given that I pointed that out explicitly in the previous comment, it's unclear where this is coming from. I'm not a disappointed justificationist, who rightly thinks we cannot find knowledge in experience and that there is no ultimate source of knowledge we can fall back on, but wrongly concludes there is no rationality, and therefore no objective distinction that can be made between the true and the false.
KF: Especially in a context dominated by self-refuting evolutionary materialist ideology. Truth as what says of what is that it is and of what is not that it is not is not going to simply go poof because some novelty demands to be taken on equal footing.
So, not only is it just novelty, but now it's self-refuting evolutionary materialist ideology? That's a mouthful. And if justification has nothing to do with it, then how is any of that even remotely relevant?
KF: Knowledge is a significant reality, for most purposes best understood as warranted credibly true belief with the degree of warrant being some level of moral certainty.
Yet, that's precisely what I'm suggesting is not the best understanding because it leaves phenomena on the table that can be explained in a unified way.
KF: In the case of say a robot embedding a computer with stored information that is causally tied to its retention, insofar as the information accessed and stored is knowledge it traces to being warranted and credibly true belief held by a designer, e.g. that stored visual data systems have been validated as accurate enough for navigation etc. KF
Yes, KF. That's an example of what you mean by Knowledge. However, you haven't shown how the definition I provided is incoherent, does not represent significant progress, etc.
PS: Why are you projecting to us a strawman justificationism?
Can you explain how your idea of warrant differs from the following? From the AI article I referenced earlier.....
For example, it is still taken for granted by almost every authority that knowledge consists of justified, true beliefs and that, therefore, an AGI’s thinking must include some process during which it justifies some of its theories as true, or probable, while rejecting others as false or improbable. But an AGI programmer needs to know where the theories come from in the first place. The prevailing misconception is that by assuming that ‘the future will be like the past’, it can ‘derive’ (or ‘extrapolate’ or ‘generalise’) theories from repeated experiences by an alleged process called ‘induction’.
After all, you did quote:
Simon Greenleaf: The true question, therefore, in trials of fact, is not whether it is possible that the testimony may be false, but, whether there is sufficient probability of its truth; that is, whether the facts are shown by competent and satisfactory evidence. Things established by competent and satisfactory evidence are said to be proved.
Popperian
July 9, 2015
July
07
Jul
9
09
2015
06:13 PM
6
06
13
PM
PDT
F/N: I should note that -- on long track record -- typically, scientific theories are explanatory models that target reliability; not truths . . . the logic of explanation brings that out. But observations linked to such need to be known to hold values with a certain reasonable reliability etc. KFkairosfocus
July 9, 2015
July
07
Jul
9
09
2015
10:40 AM
10
10
40
AM
PDT
Popperian, it is beginning to look like Lincoln's riddle: "if we call the tail of a sheep a leg how many legs does a sheep have?" The correct answer is still four, twisting words does not change realities, never mind 2 + 2 = whatever The Party needs it to be, Mr Smith. And 2 + 2 = 4 is not improved by such an exercise backed up by intimidation, torture or brainwashing etc. Dismissing knowledge and truth then pushing in novelties into those terms does not change the underlying context where such are relevant and important realities; this is not some pomo 101 where if I don't conform, my grades suffer. Especially in a context dominated by self-refuting evolutionary materialist ideology. Truth as what says of what is that it is and of what is not that it is not is not going to simply go poof because some novelty demands to be taken on equal footing. Knowledge is a significant reality, for most purposes best understood as warranted credibly true belief with the degree of warrant being some level of moral certainty. In the case of say a robot embedding a computer with stored information that is causally tied to its retention, insofar as the information accessed and stored is knowledge it traces to being warranted and credibly true belief held by a designer, e.g. that stored visual data systems have been validated as accurate enough for navigation etc. KF PS: Why are you projecting to us a strawman justificationism?kairosfocus
July 9, 2015
July
07
Jul
9
09
2015
07:58 AM
7
07
58
AM
PDT
On side note: according to materialism there is no knowledge. There is simply no knowledge in a universe with just particles in motion.
Again, epistemology is the field of philosophy that is concerned with the question, what is knowledge, how does it grow, what does it mean to say something is true, etc. As such, for a critical rationalist, such as myself...
By dissolving justificationism itself, the critical rationalist regards knowledge and rationality, reason and science, as neither foundational nor infallible, but nevertheless does not think we must therefore all be relativists. Knowledge and truth still exist, just not in the way we thought.
So, no. That is not implied by materialism. We've just made progress since then. That would only be the case if there was just one definition of knowledge, that we could never improve upon, that is not itself subject to criticism, etc. But that's like denying the entire field of epistemology exists.Popperian
July 9, 2015
July
07
Jul
9
09
2015
07:45 AM
7
07
45
AM
PDT
It is easy to construct an entity where a bloc of information that is error or noise or nonsense can play a causal role in its retention, by suitable programming for instance.
Again, based on your response, I suspect you haven't quite grasped what's being presented. Why? Your example of why that definition doesn't work simply does not work as an example of that definition. Specifically, if the program copies the block of error, rather than itself, then it is not playing a causal role in its own preservation. And if the block of error isn't doing the copying, which would necessarily be the case as you've described it as "error", then it's not playing a causal role it its own preservation either. So, yes. What you described isn't knowledge, because it doesn't fit the definition presented.
Cases in which such information are in fact knowledge will invariably meet the criteria for being so warranted as to be credibly true and treated as such — i.e. believed for good reason at some level in the origin and operation of the system. KF
Implicit in your definition is a knowing subject. Yet, I've indicated repeatedly that part of Popper's contribution is providing a universal explanation that does not depend on a knowing subject and is applicable in the case of books and even brains and genomes. This represents philosophical progress in the form of unification.Popperian
July 9, 2015
July
07
Jul
9
09
2015
07:34 AM
7
07
34
AM
PDT
Yet more: >>Neuroscience will eventually enable us to understand the mind by showing us how the brain works. But we already know enough about it to take nothing introspection tells us about the mind on trust. The phenomenon of blindsight—people who don’t have any conscious color experiences can tell the color of a thing—is enough to give us pause about the most apparently certain conclusion introspection insists on: that when you see a color you have a color experience. Then there is the fact, discovered by Libet, that actions are already determined by your brain before you consciously decide to do them! (As for determinism and the denial of real free will, that is a conclusion which, so to speak, goes without saying for scientism.) We have to add to these illusions of the will and sensory experience, robust experimental results which reveal that we actually navigate the world looking through the rear-view mirror! We don’t even see what is in front of our eyes, but continually make guesses about it based on what has worked out in our individual and evolutionary past. Discovering the illusion that we are looking through the windshield in stead of the rear view mirror, along with so much more that neuroscience is uncovering about the brain, reveals that the mind is no more a purpose-driven system than anything else in nature. This is just what scientism leads us to expect. There are no purposes in nature; physics has ruled them out, and Darwin has explained them away. Perhaps the most profound illusion introspection foists on us is the notion that our thoughts are actually recorded anywhere in the brain at all in the form introspection reports. This has to be the profoundest illusion of all, because neuroscience has been able to show that networks of human brain cells are no more capable of representing facts about the world the way conscious introspection reports than are the neural ganglia of sea slugs! The real challenge for neuroscience is to explain how the brain stores information when it can’t do so in anything like the way introspection tells us it does—in sentences made up in a language of thought. The interior monologue that introspection carries on is a sub-vocal version of the play (the tokening) of noise, ink-marks and pixels that passes for public communication. Like public speech and writing, our introspective stream of consciousness doesn’t record or report what the brain is actually doing, because the brain can’t store or manipulate information in words and sentences of any language, including mentalese. Conscious introspection is not just wrong about sensory experience, it’s no guide to cognition either. Whatever the brain does, it doesn’t operate on beliefs and wants, thoughts and hopes, fears and expectations, insofar as these are supposed to be states that “contain” sentences, and are “about” things, facts, events that are outside of the mind. That the brain no more has original intentionality than anything else does is the hardest illusion to give up, and we probably won’t be able completely to do so till neuroscience really understands the brain. Meanwhile, knowing what is not on the cards, is still enough to put in proper perspective the humanities’ endless absorption with meaning, and the persistent demands for interpretative understanding made in the human sciences. If linguistic meaning is anything at all it has got to be something like what the philosopher of language Paul Grice discovered about it: at bottom it’s a nested set of beliefs and desires that speakers have about their listeners and themselves. Grice’s own set of conditions necessary and sufficient for linguistic meaning might need to be fine- tuned. But he showed at least to a first approximation what linguistic meaning consists in. Scientism must treat this conclusion as devastating to any attempt to take semantic meaning seriously as a fact about reality. If there literally are no beliefs and desires, because the brain can’t encode information in the form of sentences, then there literally is no such thing as linguistic meaning either. It’s just a useful heuristic device, one with only a highly imperfect grip on what is going on in thought. Consequently, there is no point asking for the real, the true, the actual meaning of a work of art, or the meaning of an agent’s act, still less the meaning of a historical event or epoch. The demand of the interpretive disciplines, that we account for ideas and artifacts, actions and events, in terms of their meanings, is part of the insatiable hunger for stories with plots, narratives, and whodunits that human kind have insisted on since natural selection made us into conspiracy-theorists a half a million years ago or so. That is a taste it will be too hard to shake in everyday life. The fiction best-seller list will always be with us. But we need to move most of the works now on the non-fiction list to their rightful places among the magic realist romances, the historical and biographical novels, and the literary confessions. Nevertheless, if the mind is the brain (and scientism can’t allow that it is anything else), we have to stop taking consciousness seriously as a source of knowledge or understanding about the mind, or the behavior the brain produces. And we have to stop taking our selves seriously too. We have to realize that there is no self, soul or enduring agent, no subject of the first-person pronoun, tracking its interior life while it also tracks much of what is going on around us. This self cannot be the whole body, or its brain, and there is no part of either that qualifies for being the self by way of numerical-identity over time. >> Rosenberg is far too smart and educated not to see the number of ways this is self-refuting. We have reached reductio ad absurdum, backed up by science sez so don't listen to your doubts. Blind faith in the lab coat clad absurd. KFkairosfocus
July 9, 2015
July
07
Jul
9
09
2015
06:45 AM
6
06
45
AM
PDT
F/N: More from Rosenberg, inadvertently parallelling Plato: http://nationalhumanitiescenter.org/on-the-human/2009/11/the-disenchanted-naturalists-guide-to-reality/ >>There is no room in a world where all the facts are fixed by physical facts for a set of free floating independently existing norms or values (or facts about them) that humans are uniquely equipped to discern and act upon. So, if scientism is to ground the core morality that every one (save some psychopaths and sociopaths) endorses, as the right morality, it’s going to face a serious explanatory problem. The only way all or most normal humans could have come to share a core morality is through selection on alternative moral codes or systems, a process that resulted in just one winning the evolutionary struggle and becoming “fixed” in the population. If our universally shared moral core were both the one selected for and also the right moral core, then the correlation of being right and being selected for couldn’t be a coincidence. Scientism doesn’t tolerate cosmic coincidences. Either our core morality is an adaptation because it is the right core morality or it’s the right core morality because it’s an adaptation, or it’s not right, but only feels right to us. It’s easy to show that neither of the first two alternatives is right. Just because there is strong selection for a moral norm is no reason to think it right. Think of the adaptational benefits of racist, xenophobic or patriarchal norms. You can’t justify morality by showing its Darwinian pedigree. That way lies the moral disaster of Social Spencerism (better but wrongly known as Social Darwinism). The other alternative—that our moral core was selected for because it was true, correct or right–is an equally far fetched idea. And in part for the same reasons. The process of natural selection is not in general good at filtering for true beliefs, only for ones hitherto convenient for our lines of descent. Think of folk physics, folk biology, and most of all folk psychology. Since natural selection has no foresight, we have no idea whether the moral core we now endorse will hold up, be selected for, over the long-term future of our species, if any. This nihilistic blow is cushioned by the realization that Darwinian processes operating on our forbearers in the main selected for niceness! The core morality of cooperation, reciprocity and even altruism that was selected for in the environment of hunter-gatherers and early agrarians, continues to dominate our lives and social institutions. We may hope the environment of modern humans has not become different enough eventually to select against niceness. But we can’t invest our moral core with more meaning than this: it was a convenience, not for us as individuals, but for our genes.>> Translated, our sense of OUGHT is an illusion, so moral knowledge is tossed -- letting loose a grand delusion in our interior lives and again undermining responsible, reasonable freedom. KFkairosfocus
July 9, 2015
July
07
Jul
9
09
2015
06:38 AM
6
06
38
AM
PDT
Onlookers, all of this is not exactly news. As I have repeatedly highlighted, c 360 BC 2350+ years ago, Plato issued a clear warning about the inherently, unavoidably detrimental nature of evolutionary materialism and how its radical relativism and amorality open the door to destructive abusive nihilist radicalism:
Ath. . . .[The avant garde philosophers and poets, c. 360 BC] say that fire and water, and earth and air [i.e the classical "material" elements of the cosmos], all exist by nature and chance, and none of them by art . . . [such that] all that is in the heaven, as well as animals and all plants, and all the seasons come from these elements, not by the action of mind, as they say, or of any God, or from art, but as I was saying, by nature and chance only [ --> that is, evolutionary materialism is ancient and would trace all things to blind chance and mechanical necessity] . . . . [Thus, they hold] that the principles of justice have no existence at all in nature, but that mankind are always disputing about them and altering them; and that the alterations which are made by art and by law have no basis in nature, but are of authority for the moment and at the time at which they are made.-
[ --> Relativism, too, is not new; complete with its radical amorality rooted in a worldview that has no foundational IS that can ground OUGHT, leading to an effectively arbitrary foundation only for morality, ethics and law: accident of personal preference, the ebbs and flows of power politics, accidents of history and and the shifting sands of manipulated community opinion driven by "winds and waves of doctrine and the cunning craftiness of men in their deceitful scheming . . . " cf a video on Plato's parable of the cave; from the perspective of pondering who set up the manipulative shadow-shows, why.]
These, my friends, are the sayings of wise men, poets and prose writers, which find a way into the minds of youth. They are told by them that the highest right is might,
[ --> Evolutionary materialism -- having no IS that can properly ground OUGHT -- leads to the promotion of amorality on which the only basis for "OUGHT" is seen to be might (and manipulation: might in "spin") . . . ]
and in this way the young fall into impieties, under the idea that the Gods are not such as the law bids them imagine; and hence arise factions [ --> Evolutionary materialism-motivated amorality "naturally" leads to continual contentions and power struggles influenced by that amorality at the hands of ruthless power hungry nihilistic agendas], these philosophers inviting them to lead a true life according to nature, that is,to live in real dominion over others [ --> such amoral and/or nihilistic factions, if they gain power, "naturally" tend towards ruthless abuse and arbitrariness . . . they have not learned the habits nor accepted the principles of mutual respect, justice, fairness and keeping the civil peace of justice, so they will want to deceive, manipulate and crush -- as the consistent history of radical revolutions over the past 250 years so plainly shows again and again], and not in legal subjection to them.
We cannot say we were not warned in good time by one of the leading minds of our civilisation. Oh, you never heard of this? No prizes for guessing why. And, of course, on such a view might and manipulation make 'right' about truth too . . . so, never mind the absurdities full speed ahead. We are now beginning to reap the consequences of the folly we have sown, as we can increasingly see starting with the obvious ideologisation and disintegration of science. KFkairosfocus
July 9, 2015
July
07
Jul
9
09
2015
06:11 AM
6
06
11
AM
PDT
Onlookers, you may wonder why I am focussing on this. It is because we need to be clear just how much evolutionary materialism undermines responsible rational freedom and ends in absurdity, undermining truth, reason, knowledge, responsibility and more. KFkairosfocus
July 9, 2015
July
07
Jul
9
09
2015
05:28 AM
5
05
28
AM
PDT
F/N: Let me clip Debilis at Fide Dubitandum: https://fidedubitandum.wordpress.com/2013/04/18/atheism-and-the-inability-to-think/ >>Do you ever have thoughts about anything–or simply “think” that you do? If this strikes you as a strange question to ask in context of the debate over God’s existence, it’s likely that you haven’t read anything by prominent atheist Alex Rosenberg. He’s firmly of the opinion that our thoughts aren’t about anything at all:
Ultimately, science and scientism are going to make us give up as illusory the very thing conscious experience screams out at us loudest and longest: the notion that when we think, our thoughts are about anything at all, (The Atheist’s Guide to Reality, p. 162)
This may sound like utter nonsense (and it is). But, if you are a materialist, Rosenberg has a point. The “aboutness” of thoughts isn’t something that can be reduced to brain states alone. There is simply no way that any interaction of neurons, by itself, can objectively be about anything but itself–and nothing can be subjectively about anything without an interpreter already present. This would mean that we have to assume a mind in order to explain the mind. As such, it might “feel” as if you have thoughts about things, or thoughts at all, but (so the argument goes) this is all illusion. If you’re thinking that Rosenberg is a bit off his rocker, you’re not alone. What is an illusion after all, if it isn’t a thought? . . . . Most might think that Rosenberg has given us a beautiful reductio ad absurdum of his materialist-atheist view. If the materialism which is the core of nearly all defense of atheism breaks down into denying that thought even exists (as Rosenberg shows later in his book), well, it might seem hard to imagine anything the theist could say to make this philosophy appear more inane than it already seems. There is, however, one more thing. Rosenberg never mentions the fact that science (so beloved by him and other materialists) is founded on trust of the human capacity to think about things. If materialism leads us to reject thought altogether, it leads us to completely reject science–which depends on thought. Hence, Rosenberg’s materialism is more deeply anti-science than anything the most fundamentalist preacher ever dreamed of saying. The utter incoherence of this is striking, but there is nothing Rosenberg claims which doesn’t follow from his materialism. In this way, he’s simply being more consistent and clear-headed than most materialists. The act of rejecting the existence of anything that can’t be backed by experimental data has come around to reject itself, and science along with it . . . >> The proud attempt to capture knowledge to evolutionary materialist scientism collapses into self-falsification and epistemological-logical nihilism. Again, Haldane goes to the core of the materialist reductionism problem and the linked notion that physical facts are all the facts . . . i.e. physicalism:
“It seems to me immensely unlikely that mind is a mere by-product of matter. For if my mental processes are determined wholly by the motions of atoms in my brain I have no reason to suppose that my beliefs are true. They may be sound chemically, but that does not make them sound logically. And hence I have no reason for supposing my brain to be composed of atoms. In order to escape from this necessity of sawing away the branch on which I am sitting, so to speak, I am compelled to believe that mind is not wholly conditioned by matter.” [[“When I am dead,” in Possible Worlds: And Other Essays [1927], Chatto and Windus: London, 1932, reprint, p.209.]
It is time for a fresh start on sounder footing. KFkairosfocus
July 9, 2015
July
07
Jul
9
09
2015
05:12 AM
5
05
12
AM
PDT
Box, self-referential incoherence and self-refuting, self-falsifying absurdity. UNDERSTOOD to be such. It is quite evident that Mr Rosenberg cannot live consistent with his worldview commitments, knows that, but clings to them because of his prior commitment to evolutionary materialist scientism. In short, the matter in front of him is screaming that scientism is absurd, that rationality itself is more than brain tissue can sustain, that such is fact no 1 of our conscious existence and more but due to ideological commitment in a day dominated by scientism he clings to absurdity and invites us to do the same. Reductio ad absurdum.Epistemological nihilism fails, fails because evolutionary materialism -- never mind the lab coats -- is self-refuting. KF PS: Why not put together a draft post and submit it, I will host it.kairosfocus
July 9, 2015
July
07
Jul
9
09
2015
04:47 AM
4
04
47
AM
PDT
Kairos #38, One cannot read Rosenberg without continuously shaking one’s head in disbelief, frequent long pauses and having a good laugh from time to time — at least I cannot. To some extent Rosenberg seems to be aware of the absurdness of the consequences of materialism (to which he refers as “scientism” or “science”). However, because of his unshakeable belief in materialism those absurd consequences must nevertheless be correct.
Rosenberg: Ultimately, science and scientism are going to make us give up as illusory the very thing conscious experience screams out at us loudest and longest: the notion that when we think, our thoughts are about anything at all, inside or outside of our minds. I know this sounds absurd, but we’ll see why this must be so in the next chapter. The physical facts fix all the facts. It’s because they do that thinking about stuff is impossible. What is more, neuroscience has already made it plain how the brain thinks without its thoughts being about anything at all. Thinking about things is an overwhelmingly powerful illusion. Once we learn how this profound illusion is produced, we’ll understand why it’s hard to cast the illusions of consciousness aside for the real answers to the relentless questions about self, mind, soul, free will, and the meaning of life. But we will also see why we must do so. Chapter 8 THE BRAIN DOES EVERYTHING WITHOUT THINKING ABOUT ANYTHING AT ALL Now we are more than hallway through this tour of how science answers the persistent questions. So far, science’s challenges to common sense and to ordinary beliefs have not been difficult to accept. But the going is going to get much harder. In this chapter and the next two, we will see that several of the most fundamental things that ordinary experience teaches us about ourselves are completely illusory. [my emphasis]
The following text reads like a joke:
Rosenberg: Among the seemingly unquestionable truths science makes us deny is the idea that we have any purposes at all, that we ever make plans—for today, tomorrow, or next year. Science must even deny the basic notion that we ever really think about the past and the future or even that our conscious thoughts ever give any meaning to the actions that express them. I don’t expect you to accept these outrageous claims without compelling arguments for them. [my emphasis]
In chapter 9 Rosenberg will go on arguing that consciousness and persons don’t exist — are illusions. So, one does not exist, thoughts are not about stuff and "THE BRAIN DOES EVERYTHING WITHOUT THINKING ABOUT ANYTHING AT ALL", but Rosenberg does not expect “YOU” to accept these outrageous claims without compelling arguments for them. Wow!! Mr. Rosenberg, there are no ‘outrageous claims’ about stuff, because there are no thoughts about stuff. Secondly, there are no ‘compelling arguments’ about stuff, because there no thoughts about stuff. And finally there is no ‘person’ who can accept those non-existent outrageous claims based on non-existent compelling arguments about stuff.Box
July 9, 2015
July
07
Jul
9
09
2015
03:57 AM
3
03
57
AM
PDT
Popperian, Why did you project to me the notion that I am assuming an infallible source in knowledge? Please note again what I have repeatedly stated, that knowledge (in the soft form sense applicable to science and many other common affairs) is best summed up as warranted, credibly true belief. Where, too, the warrant will be to some level of moral certainty such that it would be irresponsible given circumstances to act as though the claim at stake were false. Such obviously includes that there are no candidate defeaters that provoke reasonable doubt that the claim is true, i.e. there is no good reason to have serious doubts about the claim accepted as credibly being true. Now, I do hold that some truths rise beyond this level and are well warranted and true beliefs that cannot be overturned, constituting hard sense knowledge. Central to these are key plumbline self-evident truths such as the triple cluster first principles of right reason: LOI, LNC, LEM. I similarly hold that it is undeniable that if a thing A exists we may ask why and investigate, drawing conclusions on reasons. (Including, possibly, insufficient information or evidence to draw firm conclusions, i.e. reasonable doubt suggesting default to the least damaging option.) Such are not dependent on claiming that there is an infallible source, they are such that if we deny them, the process of rational thought disintegrates into absurdity. Likewise, I strongly hold that if we are not responsibly free and freely reasoning, the project of rationality (as opposed to the self-referentially absurd philosophy of rationalism) collapses. This implies that any worldview position that implies the lack of such responsible freedom disintegrates into chaos. So, the issue is, are we responsibly and rationally free? For too many today the answer is that they have no worldview right to freely conclude, yes. Especially, advocates of evolutionary materialism. Now, glance back, you will see that nowhere does my chain of reasoning pivot on the assumption of a perfect oracle of truth and knowledge in general. Instead, I point to the general situation of soft form knowledge and the limited cases where we can go beyond that. Beyond, I suggest that machines that are not responsibly free, such as PCs, are not independent creators of knowledge. Their hard and software configurations may reflect knowledge, but that comes from designers. The story of the Pentium recall over math instruction problems suffices to show that infallibility is not a necessary condition of designers. And, I would put it to you that there is zero actual evidence of machines embedding FSCO/I beyond 500 - 1,000 bits being observed to come about by blind chance or mechanical necessity. Trillions of cases substantiate the origin of such cases by design, backed up by needle in haystack analysis. So, again, I find myself twisted into a strawman figure in your assertions. I suggest that it would be more productive to focus what I have actually put on the table: knowledge in the common soft form sense as warranted, credibly true belief. With warrant denoting some degree of moral certainty of capturing truth, implying no active and effective defeaters etc. KFkairosfocus
July 9, 2015
July
07
Jul
9
09
2015
02:24 AM
2
02
24
AM
PDT
Box, hold a mirror up to that argument. No argument or thoughts can be about anything, including Rosenberg's sentences. Patent self-referential absurdity and self-falsification. The pity is, those who argue like this seem blissfully unaware that they are shredding their own arguments and leave their own rationality in self-inflicted tatters. But then that is how evolutionary materialism is in so many ways. KFkairosfocus
July 8, 2015
July
07
Jul
8
08
2015
04:43 PM
4
04
43
PM
PDT
On side note: according to materialism there is no knowledge. There is simply no knowledge in a universe with just particles in motion. Moreover, thoughts, which according to materialism are just chemicals in the brain, are not about stuff.
So, when consciousness assures us that we have thoughts about stuff, it has to be wrong. The brain nonconsciously stores information in thoughts. But the thoughts are not about stuff. Therefore, consciousness cannot retrieve thoughts about stuff. There are none to retrieve. So it can’t have thoughts about stuff either. Remember, the original problem was how the Paris neurons can be about the fact that Paris is the capital of France. We simplified the problem to how they can be about Paris. The answer to that question appears to be that the Paris neurons cannot be about Paris. But we could have used the same strategy to show that they can’t be about France or about the relationship of “being the capital of” that is supposed to hold between Paris and France. In other words, the Paris neurons that carry the information that Paris is the capital of France can’t be about the fact that Paris is the capital of France. When we think that Paris is the capital of France, our thought can’t be about the fact that Paris is the capital of France. It can’t be about anything. [A.Rosenberg, ‘The Atheist’s Guide to Reality’, Ch. 8]
Knowledge/information/thoughts that are not about stuff is no knowledge/information/thoughts.Box
July 8, 2015
July
07
Jul
8
08
2015
02:11 PM
2
02
11
PM
PDT
Popperian: I note:
I note that you’re effectively saying there are good reasons to think that knowledge represents beliefs we hold for good reasons. Rather, I’m saying the complete opposite. Knowledge starts out as a guess, and becomes less wrong over time.
Not at all. I have stated that in the common and quite reasonable usage [such as science or general affairs], knowledge denotes warranted, credibly true belief. Where, warrant amounts to some degree of moral certainty. That is, unless there is solid (though not necessarily infallible) reason to hold something to be actually true, having a mere good reason to believe something is not the same as that it is knowledge. By excising truth and warrant that substantiates the credibility of what is accepted as true being actually true, I find that you have erected a strawman. Further to this, I find that there are many good reasons to hold a belief that are not shaped by it being therefore credible that what is accepted as knowledge is actually true. That is, to some degree of moral certainty, it credibly says of what is that it is, and of what is not, that it is not. Where also when something is morally certain, one would be irresponsible to act as though it were false. For instance, crossing a road in the teeth of the evidence of one's senses of sight and hearing operating under typical circumstances that a heavy vehicle is onrushing, is irresponsible. Whether one started with guesses and incrementally corrected them is of absolutely no consequence to the pivotal issue. KFkairosfocus
July 8, 2015
July
07
Jul
8
08
2015
12:19 PM
12
12
19
PM
PDT
1 2 3

Leave a Reply